Difference between revisions of "2011 AIME I Problems/Problem 2"

(Solution 2)
(started trying to draw diagram (commented out))
Line 5: Line 5:
  
 
== Solution 1 ==
 
== Solution 1 ==
 +
<!--
 +
<asy>
 +
unitsize(0.5cm);
 +
defaultpen(0.8);
 +
pair A=(0,0), B=(12,0), C=(12,10),  D=(0,10);
 +
draw(A--B--C--D--cycle);
 +
label("$A$",A,SW);
 +
label("$B$",B,SE);
 +
label("$C$",C,NE);
 +
label("$D$",D,NW);
 +
 +
pair E=(3,5), F=(9,5);
 +
dot("$E$",E,N);
 +
dot("$F$",F,N);
 +
pair G = extension(A, D, E, F);
 +
pair H = extension(B, C, E, F);
 +
draw(G--H);
 +
dot("$G$",G,W);
 +
dot("$H$", H,E);
 +
</asy>
 +
-->
  
 
Let us call the point where <math>\overline{EF}</math> intersects <math>\overline{AD}</math> point <math>G</math>, and the point where <math>\overline{EF}</math> intersects <math>\overline{BC}</math> point <math>H</math>. Since angles <math>FHB</math> and <math>EGA</math> are both right angles, and angles <math>BEF</math> and <math>DFE</math> are congruent due to parallelism, right triangles <math>BHE</math> and <math>DGF</math> are similar. This implies that <math>\frac{BH}{GD} = \frac{9}{8}</math>. Since <math>BC=10</math>, <math>BH+GD=BH+HC=BC=10</math>. (<math>HC</math> is the same as <math>GD</math> because they are opposite sides of a rectangle.) Now, we have a system:
 
Let us call the point where <math>\overline{EF}</math> intersects <math>\overline{AD}</math> point <math>G</math>, and the point where <math>\overline{EF}</math> intersects <math>\overline{BC}</math> point <math>H</math>. Since angles <math>FHB</math> and <math>EGA</math> are both right angles, and angles <math>BEF</math> and <math>DFE</math> are congruent due to parallelism, right triangles <math>BHE</math> and <math>DGF</math> are similar. This implies that <math>\frac{BH}{GD} = \frac{9}{8}</math>. Since <math>BC=10</math>, <math>BH+GD=BH+HC=BC=10</math>. (<math>HC</math> is the same as <math>GD</math> because they are opposite sides of a rectangle.) Now, we have a system:

Revision as of 09:07, 14 May 2011

Problem

In rectangle $ABCD$, $AB=12$ and $BC=10$. Points $E$ and $F$ lie inside rectangle $ABCD$ so that $BE=9$,$DF=8$,$\overline{BE}||\overline{DF}$,$\overline{EF}||\overline{AB}$, and line $BE$ intersects segment $\overline{AD}$. The length $EF$ can be expressed in the form $m\sqrt{n}-p$, where $m$,$n$, and $p$ are positive integers and $n$ is not divisible by the square of any prime. Find $m+n+p$.


Solution 1

Let us call the point where $\overline{EF}$ intersects $\overline{AD}$ point $G$, and the point where $\overline{EF}$ intersects $\overline{BC}$ point $H$. Since angles $FHB$ and $EGA$ are both right angles, and angles $BEF$ and $DFE$ are congruent due to parallelism, right triangles $BHE$ and $DGF$ are similar. This implies that $\frac{BH}{GD} = \frac{9}{8}$. Since $BC=10$, $BH+GD=BH+HC=BC=10$. ($HC$ is the same as $GD$ because they are opposite sides of a rectangle.) Now, we have a system:

$\frac{BH}{GD}=\frac{9}8$

$BH+GD=10$

Solving this system (easiest by substitution), we get that:

$BH=\frac{90}{17}$

$GD=\frac{80}{17}$

Using the Pythagorean Theorem, we can solve for the remaining sides of the two right triangles:

$\sqrt{9^2-(\frac{90}{17})^2}$ and $\sqrt{8^2-(\frac{80}{17})^2}$

Notice that adding these two sides would give us twelve plus the overlap $EF$. This means that:

$EF= \sqrt{9^2-(\frac{90}{17})^2}+\sqrt{8^2-(\frac{80}{17})^2}-12=3\sqrt{21}-12$

Since $21$ isn't divisible by any perfect square, our answer is:

$3+21+12=\boxed{36}$


Solution 2

Extend lines $BE$ and $CD$ to meet at point $G$. Draw the altitude $GH$ from point $G$ to line $BA$ extended.

$GE=DF=8$ $GB=17$

In right $\bigtriangleup GHB$, $GH=10$, $GB=17$, thus by Pythagoras Theorem we have: $HB=\sqrt{17^2-10^2}=3\sqrt{21}$

$EF=GD=3\sqrt{21}-12$

Thus our answer is: $3+21+12=\boxed{36}$


See also

2011 AIME I (ProblemsAnswer KeyResources)
Preceded by
Problem 1
Followed by
Problem 3
1 2 3 4 5 6 7 8 9 10 11 12 13 14 15
All AIME Problems and Solutions